A dth tv connection provides channels in english and other languages in the ratio 7:13. what percentage of the channels are in english

Answers

Answer 1

A DTH TV connection provides channels in English and other languages in the ratio 7:13. To find out what percentage of the channels are in English, you need to divide the number of English channels by the total number of channels and then multiply the result by 100.

Let's assume that there are a total of 100 channels available on this DTH TV connection. According to the given ratio, 7 out of every 20 channels will be in English. So, the percentage of channels in English will be:

(7/20) x 100 = 35%

Therefore, 35% of the channels on this DTH TV connection are in English.

To know more about TV connection refer here

https://brainly.com/question/30629673#

#SPJ11


Related Questions

Of 100 random students surveyed, 42 own a dog, 34 own a cat, 15 own a dog and a cat, and 9 own neither a dog nor a cat. Based upon the results, how many of the next 20 students surveyed would you expect to own a dog and a cat?

Answers

In the next 20 students surveyed, you would expect 5 to own a dog and a cat

How many of the next 20 students surveyed would you expect to own a dog and a cat?

From the question, we have the following parameters that can be used in our computation:

Dog = 42

Cat = 34

Dog and cat = 15

Neither = 9

This means that

P(Dog and cat) = 15/100

When evaluated, we have

P(Dog and cat) = 5/20

So, when the next 20 students surveyed, we have

Dog and cat = 5/20 * 20

Evaluate

Dog and cat = 5

Hence, the number of dogs and cats is 5

Read more about expected value at

brainly.com/question/15858152

#SPJ1

Given the following joint PDF function of two continuous random variables x and y :

[tex]f(x,y) = \left \{ {{1/4x^2 +1/4y^2 +1/6xy} \atop {0}} \right. 0\leq x\leq 1 ; 0\leq y\leq 2[/tex]\


a) find the distribution function F(x,y)

b) find marginal PDF for f(x) and f(y)

c) find P ( 0[tex]0\leq x\leq 1/2 , 0\leq y\leq 1/2[/tex]

d) if u= 2x-y and v = -x+y find the dense joint density function of u and v

Answers

A. The distribution function F(x,y) is ¹¹/₁₈ + ¹/₁₂ x² - ¹/₁₈ y² + ¹/₁₂ xy

B. The marginal PDF of x is ¹/₂x + ¹/₆ + ¹/₁₂x² for 0≤x≤1 and for y is /₂y + ¹/₆ + ¹/₁₂y² for 0≤y≤2

C. P(0≤x≤1/2, 0≤y≤1/2) is ¹/₃₂ + ¹/₉₆ x² for 0≤x≤1/2

D. The joint PDF of u=2x-y and v=-x+y is f(u,v) = (1/27)(2u^2+2v^2-2uv)

How did we get these values?

a) To find the distribution function F(x,y), integrate the joint PDF over the appropriate limits.

F(x,y) = ∫∫f(u,v)dudv

The limits of integration are not specified, so, determine them from the limits of the variables x and y.

So,

F(x,y) = ∫∫f(u,v)dudv

= ∫∫f(x+y,x-y)dudv (substituting u = x+y and v = x-y)

= ∫∫(¹/₄(u²+v²)+¹/₆(u²-v²))dudv (substituting x and y back in terms of u and v)

The limits of integration for u and v can be found by solving for u and v in terms of x and y as follows:

u = x+y

v = x-y

x = (u+v)/2

y = (u-v)/2

0 ≤ x ≤ 1; 0 ≤ y ≤ 2

implies

0 ≤ (u+v)/2 ≤ 1; 0 ≤ (u-v)/2 ≤ 2

Solving the above inequalities gives the following limits:

0 ≤ u ≤ 2; -u ≤ v ≤ u;

Thus,

F(x,y) = ∫∫(¹/₄(u²+v²)+¹/₆(u²-v²))dudv

= ∫²₀ ∫ᵘ_(-u) (1/4(u²+v²)+¹/₆(u²-v²))dvdu

= ¹¹/₁₈ + ¹/₁₂ x² - ¹/₁₈ y² + ¹/₁₂ xy

b) To find the marginal PDF of x, integrate the joint PDF over all possible values of y:

f(x) = ∫f(x,y)dy

So,

f(x) = ∫²₀ (¹/₄x + ¹/₄y²/x + ¹/₆y) dy

= ¹/₂x + ¹/₆ + ¹/₁₂x² for 0≤x≤1

In the same way, find the marginal PDF of y, by integrating the joint PDF over all possible values of x:

f(y) = ∫f(x,y)dx

So,

f(y) = ∫¹₀ (¹/₄x²/y + ¹/₄y + ¹/₆xy) dx

= ¹/₂y + ¹/₆ + ¹/₁₂y² for 0≤y≤2

c) To find P(0≤x≤1/2, 0≤y≤1/2), integrate the joint PDF over the appropriate limits:

P(0≤x≤1/2, 0≤y≤1/2) = ∫∫f(x,y)dxdy

So,

P(0≤x≤1/2, 0≤y≤1/2) = ∫¹₀ ∫^(1/2)_0 (¹/₄x² + ¹/₄y²/x + ¹/₆xy) dydx

= ¹/₃₂ + ¹/₉₆ x² for 0≤x≤1/2

d) To find the joint PDF of u=2x-y and v=-x+y, express x and y in terms of u and v and then apply transformation formula.

From the given equations, solve for x and y in terms of u and v as follows:

x = (u+v)/3

y = (v-u)/3

Now, find the Jacobian of the transformation:

J = ∂(x,y)/∂(u,v) =

| ∂x/∂u ∂x/∂v |

| ∂y/∂u ∂y/∂v |

=

| 1/3 1/3 |

| -1/3 1/3 |

So, |J| = 2/9

Using the transformation formula for joint PDFs:

f(u,v) = f(x(u,v), y(u,v)) |J|

Substituting x and y in terms of u and v:

f(u,v) = f((u+v)/3, (v-u)/3) (2/9)

Substituting the given joint PDF for f(x,y), we get:

f(u,v) = (¼((u+v)/3)² + ¼((v-u)/3)² + ⅙((u+v)/3)((v-u)/3))(2/9)

Simplify:

f(u,v) = (1/27)(2u²+2v²-2uv)

So, the joint PDF of u=2x-y and v=-x+y is:

f(u,v) = (1/27)(2u²+2v²-2uv)

learn more about distribution function: https://brainly.com/question/25301796

#SPJ4

Find the exact location of all the relative and absolute extrema of the function. (Order your answers from smallest to largest x.) g(x) = 3x³ - 36x with domain [-4, 4] g has an absolute minimum at (x,y) =

Answers

We can see that the absolute minimum occurs at (x, y) = (2, -48).

To find the relative and absolute extrema of the function g(x) = 3x³ - 36x on the domain [-4, 4], we first need to find the critical points. We do this by finding the first derivative, setting it to zero, and solving for x.

g'(x) = d(3x³ - 36x)/dx = 9x² - 36

Setting g'(x) to 0:

0 = 9x² - 36
x² = 4
x = ±2

These are our critical points. To determine if these are minima, maxima, or neither, we use the second derivative test.

g''(x) = d(9x² - 36)/dx = 18x

At x = -2:
g''(-2) = -36 < 0, so it's a relative maximum.

At x = 2:
g''(2) = 36 > 0, so it's a relative minimum.

Now, we need to compare the function values at the critical points and endpoints of the domain to determine the absolute extrema.

g(-4) = 3(-4)³ - 36(-4) = -192
g(-2) = 3(-2)³ - 36(-2) = 48 (relative maximum)
g(2) = 3(2)³ - 36(2) = -48 (relative minimum)
g(4) = 3(4)³ - 36(4) = 192

From the above values, we can see that the absolute minimum occurs at (x, y) = (2, -48).

To learn more about absolute minimum, refer below:

https://brainly.com/question/29030328

#SPJ11

What is the value of X in circle O below?
Need help on all step by step preferably

Answers

Answer:

a. x = 68

b. x = 55

c. x = 18

Step-by-step explanation:

Formula

Inscribed angle = Central angle/2

a.

x = 136/2

x = 68

b.

x = ( 360 - 150 - 100 )/2

= 110/2

x = 55

c.

x = 18

Miles is buying a new rain barrel to help with his watering problem. the rain barrel is shaped like a right circular cylinder. what is the volume of the rain barrel if it is 27 inches tall and has a diameter of 22 inches. use 3.14 for pi.

Answers

The volume of the rain barrel is approximately 10,256.58 cubic inches.

To get the volume of the rain barrel, which is shaped like a right circular cylinder, you need to use the formula for the volume of a cylinder: V = πr²h. Here, V represents the volume, r is the radius, and h is the height of the cylinder.
The given diameter of the rain barrel is 22 inches. To find the radius (r), you need to divide the diameter by 2:
r = 22 / 2 = 11 inches.
The height (h) of the rain barrel is given as 27 inches.
Now, you can plug these values into the formula and use 3.14 for pi (π):
V = πr²h
V = 3.14 * (11²) * 27
V = 3.14 * (121) * 27
V = 3.14 * 3267
V ≈ 10,256.58 cubic inches
So, the volume of the rain barrel is approximately 10,256.58 cubic inches.

Learn more about volume here, https://brainly.com/question/1972490

#SPJ11

Sam has a box shaped like a rectangular prism. It measures 1/6 inches in height, 1/3 in. Wide and 1/2 in. Long. What is the volume of the box? Leave your answer as an improper fraction

Answers

A rectangular prism is a three-dimensional shape with six faces, all of which are rectangles. The volume of a rectangular prism can be found by multiplying the length, width, and height of the prism.

In this case, Sam's box has a height of 1/6 inches, a width of 1/3 inches, and a length of 1/2 inches. To find the volume, we need to multiply these three dimensions:

(1/6) x (1/3) x (1/2) = 1/36 cubic inches.

Therefore, the volume of Sam's box is 1/36 cubic inches, which is an improper fraction because the numerator is greater than the denominator.

To know more about rectangular prism refer here

https://brainly.com/question/21308574#

#SPJ11

The regular polygon has the following measures.
a = 2√3 yd
s = 4 yd
Segment a is drawn from the center of the polygon
perpendicular to one of its sides.
What is the vocabulary term for segment a?
What is the area of the polygon?
Round to the nearest tenth and include correct units.
Show all your work.

Answers

The vocabulary for the segment a is the apothem

The area of the polygon is about 41.6 yd²

What is the area of a regular figure?

The area of a regular figure is the extent of the planer space the figure occupies.

The length of each side of the regular polygon, s = 4 yd

The length of the segment a = 2·√3

The vocabulary term for the segment a drawn from from the center of the polygon and perpendicular to one of its sides is the apothem

Therefore, the vocabulary term for segment a is the apothem

The polygon is a hexagon.

The area of a hexagon is; A = ((3·√3)/2) × s²

Therefore, the area of the polygon is; A = ((3·√3)/2) × (4)² = 24·√3 ≈ 41.6

The area of the polygon is about 41.6 yd²

Learn more on the area of a polygon here: https://brainly.com/question/17046879

#SPJ1

Create trig ratios for sin, cos, and tan:

Answers

You can look the image.

It is very clear.

Answer:

[tex]\sin (Z)=\sf\dfrac{9}{15}[/tex]        [tex]\cos (Z)=\sf\dfrac{12}{15}[/tex]        [tex]\tan(Z)=\sf \dfrac{9}{12}[/tex]

Step-by-step explanation:

To create trigonometric ratios for angle Z in the given right triangle XYZ, we can use the trigonometric ratios.

[tex]\boxed{\begin{minipage}{9.4 cm}\underline{Trigonometric ratios} \\\\$\sf \sin(\theta)=\dfrac{O}{H}\quad\cos(\theta)=\dfrac{A}{H}\quad\tan(\theta)=\dfrac{O}{A}$\\\\where:\\ \phantom{ww}$\bullet$ $\theta$ is the angle. \\ \phantom{ww}$\bullet$ $\sf O$ is the side opposite the angle. \\\phantom{ww}$\bullet$ $\sf A$ is the side adjacent the angle. \\\phantom{ww}$\bullet$ $\sf H$ is the hypotenuse (the side opposite the right angle). \\\end{minipage}}[/tex]

From inspection of the right triangle XYZ:

θ = ZO = XY = 9A = YZ = 12H = XZ = 15

Substitute these values into the three ratios to create the trigonometric ratios for angle Z:

[tex]\sin (Z)=\sf \dfrac{O}{H}=\dfrac{9}{15}[/tex]

[tex]\cos (Z)=\sf \dfrac{A}{H}=\dfrac{12}{15}[/tex]

[tex]\tan(Z)=\sf \dfrac{O}{A}=\dfrac{9}{12}[/tex]

if loga 3=p and log 5=q then loga 45 is equivalent to which of the following:
QP^2, Q+2P, 5Q+3P, 3(Q+P), or 3(Q+2P)

Answers

the answer to log 45 will be Q + 2P.

What is Logarithmic functions?

A logarithmic function is a type of function that can be expressed in the form: f(x) = a log(bx + c) + d where a, b, c, and d are constants and x is the independent variable. The base of the logarithm is usually assumed to be 10, but can be any other positive number.

Logarithmic functions are used in a variety of applications, including finance, physics, and engineering. In finance, logarithmic functions are used to calculate compound interest. In physics, logarithmic functions are used to describe exponential decay and growth. In engineering, logarithmic functions are used to model the behavior of electrical circuits and other systems.

log a (45) = log a (9) + log a (5)

Next, we can use the fact that log a (x^n) = n log a (x) to simplify the first term:

log a (9) = log a (3²) = 2 log a (3) = 2p

Finally, we can substitute the given values for p and q and simplify the expression:

log a (45) = 2p + q = 2 log a (3) + log a (5) = log a (3²) + log a (5) = log a (3² * 5)

Therefore, we have:

log a (45) = log a (3² * 5)

Now, using the property that log a (x * y) = log a (x) + log a (y), we can simplify this expression even further:

log a (45) = log a (3²) + log a (5) = 2 log a (3) + log a (5) = Q + 2P

Therefore, log a (45) is equivalent to Q + 2P.

Therefore, the answer is Q + 2P.

Learn more about Logarithmic functions, by the following link.

https://brainly.com/question/30194309

#SPJ1

Rewrite the polynomial 2x^2+x^3+-7x+1 in standard form. Show your steps

Answers

So the polynomial 2x² + x³ - 7x + 1 in standard form is x³ + 2x² - 7x + 1.

What is the polynomial?

To rewrite the polynomial 2x² + x³ - 7x + 1 in standard form, we need to write the terms in descending order of degree.

So we start with the highest degree term:

Then we add the next highest degree term: 2x²

Followed by the next highest degree term: -7x

Finally, we add the constant term: +1

Putting all the terms together, we get:

x³ + 2x² - 7x + 1

So the polynomial 2x² + x³ - 7x + 1 in standard form is x³ + 2x² - 7x + 1.

Learn more about polynomial from

https://brainly.com/question/2833285

#SPJ1

Its linear equation world problems please help asap also do them step by step i need the equation also

the three angles of a triangle are

(2x +5) ⃘

(2x +5) ⃘


,

(x −10) ⃘ and 65 ⃘

(x −10) ⃘ and 65 ⃘

calculate the size of each angle.




determine three consecutive odd numbers whose sum is 33.




determine three consecutive even numbers whose sum is 102.

Answers

The size of three angles of the triangle are 55 degrees, 55 degrees, and 15 degrees. The three consecutive odd numbers are 9, 11, and 13 and three consecutive even numbers are 32, 34, and 36.

1.To find the size of each angle in the triangle, we know that the sum of all angles in a triangle is 180 degrees. So we can set up an equation:

(2x + 5) + (2x + 5) + (x - 10) + 65 = 180

Simplifying and solving for x, we get:

5x + 55 = 180

5x = 125

x = 25

Now we can substitute x back into the expressions for each angle and simplify:

2x + 5 = 55 degrees

2x + 5 = 55 degrees

x - 10 = 15 degrees

Therefore, the three angles of the triangle are 55 degrees, 55 degrees, and 15 degrees.

2. Let's call the first odd number x. Then the next two consecutive odd numbers would be x + 2 and x + 4. We know that the sum of these three numbers is 33, so we can set up an equation:

x + (x + 2) + (x + 4) = 33

Simplifying and solving for x, we get:

3x + 6 = 33

3x = 27

x = 9

Therefore, the three consecutive odd numbers are 9, 11, and 13.

3. Let's call the first even number x. Then the next two consecutive even numbers would be x + 2 and x + 4. We know that the sum of these three numbers is 102, so we can set up an equation:

x + (x + 2) + (x + 4) = 102

Simplifying and solving for x, we get:

3x + 6 = 102

3x = 96

x = 32

Therefore, the three consecutive even numbers are 32, 34, and 36.

To know more about triangles refer here:

https://brainly.com/question/2773823

#SPJ11

Choose the system for the graph.

Answers

The system of inequalities in the graph is the one in option A.

y ≥ (-2/5)x - 2/5

y ≥ (3/2)*x - 1

Which is the system of inequalities in the graph?

Here we can see the graph of a system of inequalities, on the graph we can see two lines.

The first one is a line with a positive slope, it has an y-intercept of -1, the shaded region is above that line,  and it is a solid line, so one of the inequalities is:

y ≥ a*x - 1

Where a is positive.

The second line has a negative slope, and we can see that the shaded region is also above the line, so this second inequality is like:

y ≥ line with negative slope.

It is easy to identify the correct option because there is only one with these properties, which is the first option:

y ≥ (-2/5)x - 2/5

y ≥ (3/2)*x - 1

LEarn moer about systems of inequalities at:

https://brainly.com/question/9774970

#SPJ1

The table shows the number of cups of flour, f, that a bakery needs for the number of pound cakes that they make, p.



Pound Cakes, p 3 6 9 14


Cups of Flour, f 8. 25 16. 5 24. 75 ?



Part A


Which equation relates the number of cups of flour to the number of pound cakes that the bakery makes?


f = 2. 75p


f = 0. 343p


f = 2. 75p + 8. 25


f = 0. 343p + 16. 5


Part B


How many cups of flour are needed for 14 cakes?


4. 802


21. 302


38. 5


46. 75

Answers

A) The equation relates the number of cups of flour to the number of pound cakes that the bakery makes is f = 2.75p

B) Cups of flour are needed for 14 cakes is 38.5

A) The number of cups of flour, f, that a bakery needs for the number of pound cakes that they make, p is directly proportional to each other which can be written in form ,

f/p = 8.25/3

f = (8.25/3)×p

f = 2.75 p

The equation forms is f = 2.75p

B) Cups of flour are needed for 14 cakes

Here p = 14

by putting the value in the equation we get ,

f = 2.75(14)

f = 38.5

hence , cups of flour are needed for 14 cakes is 38.5

To know more about equation click here :

https://brainly.com/question/24169758

#SPJ4

Gabby had 578 yards of fabric. she used 3215 yards of fabric. estimate the amount of fabric gabby has left. 1 yard 2 yards 3 yards 4 yards

Answers

Gabby does not have any fabric left, as she used more fabric than she had to start with.

How much fabric does Gabby have left after using 3215 yards, and what is the estimate?

Based on the information given, Gabby started with 578 yards of fabric and used 3215 yards of fabric. To estimate the amount of fabric Gabby has left, we need to subtract the amount of fabric used from the starting amount of fabric:

578 yards - 3215 yards = -2637 yards

Since the result is a negative number, it doesn't make sense in this context. It's possible that there was a mistake in the numbers given, or Gabby used more fabric than she had to start with.

Without further information or clarification, we cannot estimate the amount of fabric Gabby has left.

To estimate the amount of fabric Gabby has left, we can subtract the amount of fabric she used from the amount of fabric she had initially.

So, to find the estimate for the amount of fabric Gabby has left, we can perform the following calculation:

Estimate for the amount of fabric Gabby has left = 578 yards (initial amount) - 3215 yards (amount used)

Estimate for the amount of fabric Gabby has left = -2637 yards

However, the result is negative, which means that Gabby doesn't have any fabric left, and she needs to purchase an additional 2637 yards to make up for the shortfall.

Therefore, the estimate for the amount of fabric Gabby has left is 0 yards (she needs to purchase more fabric to continue her work).

Find the indicated real nth root(s) of a. n=3, a=27

Answers

The indicated real nth root(s) of a, where n=3 and a=27 is 3.


You need to find the indicated real nth root(s) of a, where n=3 and a=27. In other words, you need to find the real number(s) that, when raised to the power of 3, equal 27.

Here's a step-by-step explanation:

1. Identify the given values: n=3 and a=27.
2. Write the equation: x^n = a, where x is the real nth root you're trying to find.
3. Substitute the given values: x^3 = 27.
4. Solve the equation for x: x = 3, since 3^3 = 27.

Your answer is x = 3, which is the real 3rd root of 27.

Learn more about indicated real nth root(s),

https://brainly.com/question/30375320

#SPJ11

400 people attended a concert 10% of the people came from Scotland 25% of the people came form Wales How many more pepole came from Wales than Scotland

Answers

If 400 people attended a concert 10 percent of the people came from Scotland 25 percent of the people came form Wales, there were 60 more people from Wales than from Scotland.

To find out how many more people came from Wales than Scotland at a concert with 400 attendees, we'll first calculate the number of people from each region.

1. Determine the number of people from Scotland:
Since 10% of the people came from Scotland, we'll multiply the total attendees (400) by 10% (0.10).
400 * 0.10 = 40 people from Scotland.

2. Determine the number of people from Wales:
Since 25% of the people came from Wales, we'll multiply the total attendees (400) by 25% (0.25).
400 * 0.25 = 100 people from Wales.

3. Calculate the difference between the number of attendees from Wales and Scotland:
Subtract the number of people from Scotland (40) from the number of people from Wales (100).
100 - 40 = 60 more people from Wales than Scotland.

In conclusion, at the concert with 400 attendees, there were 60 more people from Wales than from Scotland.

To know more about percent, visit:

https://brainly.com/question/30908449#

#SPJ11

a car drives 10.5 miles in 1/6 hour. what is its speed in miles per hour

Answers

Answer:

(Credit to guy/girl above) 63 miles 10 1/2 x 6 is 63.

Step-by-step explanation:

pls mark brainliest

How do you do this problem?

Answers

Knowing that tan(x) = 3/5 and using a trigonometric identity, we will get that:

tan(2x) = 1.875

How to find the tangent of 2x?

There is a trigonometric identity we can use for this, we know that:

[tex]tan(2x) = \frac{2tan(x)}{1 - tan^2(x)}[/tex]

So we only need to knos tan(x), which we already know that is equal to 3/5, then we can replace it in the formula above to get:

[tex]tan(2x) = \frac{2*3/5}{1 - (3/5)^2}\\\\tan(2x) = \frac{6/5}{1 - 9/25} \\tan(2x) = 1.875[/tex]

That is the value of the tangent of 2x.

Learn more about the tangent function at:

https://brainly.com/question/1533811

#SPJ1

How many more cubic inches of popcorn does the jumbo size hold compared to the regular size round to the nearest tenth

Answers

Cubic inches of popcorn does the jumbo size hold compared to the regular size round to the nearest tenth

To determine how many more cubic inches of popcorn the jumbo size holds compared to the regular size, you would need to:

1. Find the volume (in cubic inches) of both the jumbo and regular size popcorn containers.
2. Subtract the volume of the regular size container from the volume of the jumbo size container.

Unfortunately, without specific dimensions for the jumbo and regular size popcorn containers, I cannot provide a numerical answer. Please provide the dimensions, and I would be happy to help you with the calculations.

Learn more about Cubic inches https://brainly.com/question/31902919

#SPJ11

Lorena es una estudiante que utiliza una red social cada 8 días. Su amigo Luis accede cada 6 días y su hermana Alexa ingresa cada 10 días. Si ellos coincidieron en su visita a esta red social el día 24 de julio

Answers

The next time they will coincide is on November 21. when Lorena uses a social network every 8 days, Luis logs in every 6 days, and his sister and Alexa log in every 10 days.

To find the time when all three coincided, we need to find the least common multiple (LCM) of 6, 8, and 10. The LCM of 6, 8, and 10 is given as,

6 8 10 | 2

3  4  5 | 3

1   4  5 | 4

1   1  5 | 5

1   1   1

LCM = 2 × 3 × 4 × 5 = 120

if they coincided on July 24, To find the time when all three coincided we need to add 120 days to July 24 to find the next time they will coincide.  if we add 120 days to July 24 we will get the result as November 21.

Therefore, The next time they will coincide is on November 21.

To learn more about least common multiple (LCM):

https://brainly.com/question/29635300

#SPJ4

The question is,

Lorena is a student who uses a social network every 8 days. His friend Luis logs in every 6 days and his sister Alexa logs in every 10 days. If they coincided with their visit to this social network on July 24 when will they coincide next time?

A manufacturer measures the number of cell phones sold using the binomial 0. 015c+2. 81. She also measures the wholesale price on these phones using a binomial 0. 011c+3. 52. Calculate her revenue if she sells 100,000 cell phones. Revenue = (numberofcellphones)(wholesaleprice) = (0. 015c+2. 81)(0. 011c+3. 52)

Answers

When the manufacturer sells 100,000 cell phones, her revenue will be approximately $1,657,993.39.


To find the revenue for selling 100,000 cell phones, we will first evaluate both binomials for the given number of cell phones (c = 100,000) and then multiply them together.

Step 1: Evaluate the first binomial (number of cell phones sold) for c = 100,000:
0.015c + 2.81 = 0.015(100,000) + 2.81 = 1,500 + 2.81 = 1,502.81

Step 2: Evaluate the second binomial (wholesale price) for c = 100,000:
0.011c + 3.52 = 0.011(100,000) + 3.52 = 1,100 + 3.52 = 1,103.52

Step 3: Calculate the revenue by multiplying the results of the two binomials:
Revenue = (1,502.81)(1,103.52) = 1,657,993.3912

So, when the manufacturer sells 100,000 cell phones, her revenue will be approximately $1,657,993.39. This calculation is based on the binomial expressions provided for the number of cell phones sold (0.015c+2.81) and the wholesale price (0.011c+3.52).

To know more about revenue, visit:

https://brainly.com/question/31683012#

#SPJ11

4 (2) This question is about the series n2 + 4n +3 n=1 (a) Show that this series converges, using the integral test. (Hint: Partial fraction decomposition.) (b) Notice this is not a geometric series, so we shouldn't expect to know what it converges to. But use the decomposition 4 into the difference n2 4n of two sums. (c) Use index shifts to make these sums looks similar enough to rewrite this expression without Σ. 4 (d) Take the limit as B+ 0 to find n2 + 4n +3 B from part (a) to break m2 + An + 3 n=1 n=1 (2) 10

Answers

(a) Given: f(x) = x^2 + 4x + 3.

The partial fraction decomposition of f(x) is:

f(x) = (x+1)(x+3)

Now, we need to find the integral of this function from 1 to infinity:

∫[1,∞] (x+1)(x+3) dx

Since the integral converges, we can conclude that the series also converges.

(b) This series is not geometric, so we don't know what it converges to. However, we can decompose the given series as the difference of two sums:

Σ(n^2 + 4n + 3) = Σ(n^2) - Σ(4n)

(c) We can use index shifts to make these sums look similar enough to rewrite the expression without Σ:

Σ(n^2) - Σ(4n) = Σ(n^2 - 4n)

(d) To find the limit as B approaches 0, we can evaluate the limit of the expression n^2 + 4n + 3:

lim(B→0) (n^2 + 4n + 3) = n^2 + 4n + 3

So, the limit of the series is n^2 + 4n + 3.

9. the square footage and monthly rental of 15 similar one-bedroom apartments yield the linear


regression formula y = 1.3485x + 840.51, where x represents the square footage and y represents


the monthly rental price. round answers to the nearest whole number.

Answers

Based on the linear regression formula y = 1.3485x + 840.51, you can calculate the monthly rental price (y) for a one-bedroom apartment by plugging in the square footage (x) of the apartment.

The linear regression formula for the 15 similar one-bedroom apartments is y = 1.3485x + 840.51, where x represents the square footage and y represents the monthly rental price. This means that for every square foot increase in the apartment size, the monthly rental price is predicted to increase by $1.35.

The y-intercept of the formula is $840.51, which represents the predicted monthly rental price for an apartment with 0 square footage (this is not possible in reality, but is used in the formula for mathematical purposes). To get the rental price, round your answer to the nearest whole number. For example, if an apartment has 500 square feet, you'd calculate: y = 1.3485(500) + 840.51 ≈ 1344.76, which rounds to $1,345 as the monthly rental price.

More on linear regression: https://brainly.com/question/29665935

#SPJ11

Using technology, calculate the weighted mean of the RORs for each portfolio. Based on the results, which list shows a comparison of the overall performance of the portfolios, from best to worst?

Answers

The ranking from worst to best is Portfolio 3, Portfolio 2, and Portfolio 1. So, correct option is B.

To calculate the weighted mean of the RORs for each portfolio, we need to first multiply each ROR by the corresponding portfolio value and then sum the products for each portfolio. We then divide the total by the sum of the portfolio values.

The weighted mean for Portfolio 1 = [(10.4% x $700) + (-29.7% x $12,000) + (37.2% x $600) + (7.5% x $4,400) + (6.3% x $250)] / ($700 + $12,000 + $600 + $4,400 + $250) = -16.8%

Similarly, the weighted mean for Portfolio 2 = 3.8% and for Portfolio 3 = 11.2%.

Based on the results, the list that shows a comparison of the overall performance of the portfolios from worst to best is option b) Portfolio 3, Portfolio 2, Portfolio 1. Portfolio 3 has the highest weighted mean return of 11.2%, followed by Portfolio 2 with a return of 3.8%, and Portfolio 1 has the lowest weighted mean return of -16.8%.

Therefore, correct option is B.

To learn more about weighted mean click on,

https://brainly.com/question/30168285

#SPJ1

Complete question is:

ROR     Portfolio 1     Portfolio 2     Portfolio 3

10.4%        $700              $6,000         $3,500

-29.7%    $12,000           $9,000         $5,500

37.2%        $600              $4,500         $5,750

7.5%          $4,400            $2,000        $1,500

6.3%          $250                $1,100          $4,500

Calculate the weighted mean of the RORs for each portfolio. Based on the results, which list shows a comparison of the overall performance of the portfolios, from worst to best?

a) Portfolio 3, Portfolio 1, Portfolio 2

b) Portfolio 3, Portfolio 2, Portfolio 1

c) Portfolio 1, Portfolio 2, Portfolio 3

d) Portfolio 1, Portfolio 3, Portfolio 2

pyramid A and pyramid B are similar. find the surface area of pyramid B to the nearest hundredth.

Answers

The surface area of pyramid B to the nearest hundredth is 58.67 cm²

What are similar figures?

Similar figures are two figures having the same shape. The ratio of the corresponding sides of similar shapes are equal.

The scale ratio of the height of the pyramid A to B is

9/6 = 3/2

Area factor = (3/2)² = 9/4

9/4 = 132/x

9x = 132×4

9x = 528

divide both sides by 9

x = 528/9

x = 58.67cm²

Therefore the surface area of pyramid B is 58.67cm².

learn more about similar figures from

https://brainly.com/question/28719932

#SPJ1

A ring-shaped region is shown below.
Its inner radius is 9m, and its outer radius is 13m.
Find the area of the shaded region.
Use 3.14 for Pie. Do not round your answer.

Answers

The area of the ring-shaped region with radii of 9m and 13m is approximately 276.32 square meters.

What is Area?

The area is the region defined by an object's shape. The area of a shape is the space covered by a figure or any two-dimensional geometric shape in a plane.

What is Perimeter?

The perimeter of a shape is defined as the total distance surrounding the shape. It is the length of any two-dimensional geometric shape's outline or boundary.

According to the given information:

The given shape is a two concentric circles with radii of 9m and 13m, we can calculate the area of this region using the formula for the area of a circle:

Area of shaded region = Area of outer circle - Area of inner circle

The area of a circle is given by the formula A = πr^2, where r is the radius of the circle.

Area of inner circle = π(9)^2 = 81π

Area of outer circle = π(13)^2 = 169π

Area of shaded region = 169π - 81π = 88π

Using the value of π = 3.14, we get:

Area of shaded region = 88π = 88(3.14) = 276.32 square meters (rounded to two decimal places)

Therefore, the area of the ring shaped region with radii of 9m and 13m is approximately 276.32 square meters.

To know more about Area visit : https://brainly.com/question/1631786

#SPJ1

Which number line shows the sum of -8, 4, and -2?
o
a +++++
-15
10
-5
0
5
10
15
b the
- 15
- 10
-5
0
15
10
15
o
chef
15
-10
5
0
5
10
15
o
d
-15
-10
0
5
110
15

Answers

Add the given numbers: -8 + 4 + (-2) = -6. So, the sum of -8, 4, and -2 is -6.

Which number line shows the sum of -8, 4, and -2?

To represent -6 on a number line, we need to find its position relative to zero. Since -6 is negative, it will be located to the left of zero. We count 6 units to the left of zero on the number line to represent -6. Therefore, the number line that shows the sum of -8, 4, and -2 is:

o----+----+----+----+----+----+----+----+----+----o

-15 -10 -5 0 5 10 15 20 25 30

-6

So, the complete answer is:

The sum of -8, 4, and -2 is -6.

To represent -6 on a number line, locate 6 units to the left of zero.

Learn more about number line",

brainly.com/question/13425491

#SPJ11

What is the area that has 160ft tall 100 feet wide and another area that has 60ft long and 40ft wide , add both shapes together

Answers

The area for the first shape is  16,000 square feet, the area for the second shape is 2,400 square feet. The total area of both shapes added together is 18,400 square feet.

To find the area of the first shape, which is a rectangle that is 160 feet tall and 100 feet wide, we can use the formula:

Area = length x width

So, for the first shape, the area is:

Area = 160 ft x 100 ft

Area = 16,000 square feet

To find the area of the second shape, which is a rectangle that is 60 feet long and 40 feet wide, we can use the same formula:

Area = length x width

So, for the second shape, the area is:

Area = 60 ft x 40 ft

Area = 2,400 square feet

To find the total area of both shapes added together, we simply add the two areas:

Total Area = 16,000 square feet + 2,400 square feet

Total Area = 18,400 square feet

Therefore, the total area of both shapes added together is 18,400 square feet.

Learn more about the area;

brainly.com/question/1658516

#SPJ11

Debra has these snacks from a birthday party in a bag.



4 bags of chips


5 fruit snacks


6 chocolate bars


3 pieces of bubble gum



Debra will randomly choose one snack from the bag. Then she will put it back and randomly choose another snack. What is the probability that she will choose a chocolate bar and then a piece of gum?



A. 1/2


B. 1/3


C. 1/9


D. 1/18

Answers

Your answer is D. 1/18  is the probability that she will choose a chocolate bar and then a piece of gum


First, let's determine the total number of snacks in the bag:
4 bags of chips + 5 fruit snacks + 6 chocolate bars + 3 pieces of bubble gum = 18 snacks

Next, let's find the probability of choosing a chocolate bar:
There are 6 chocolate bars and 18 snacks total, so the probability is 6/18, which simplifies to 1/3.

Since she puts the chocolate bar back, the total number of snacks remains the same. Now, let's find the probability of choosing a piece of gum:
There are 3 pieces of gum and 18 snacks total, so the probability is 3/18, which simplifies to 1/6.

Finally, to find the probability of both events happening, multiply the probabilities together:
(1/3) * (1/6) = 1/18

So, the probability that Debra will choose a chocolate bar and then a piece of gum is 1/18. Your answer is D. 1/18.

learn more about "Probability":-https://brainly.com/question/25870256

#SPJ11

what fraction is less greater than 1/2 and less than 4/5

Answers

A fraction that is greater than 1/2 and less than 4/5, we need to consider fractions between these two values. A fraction that is greater than 1/2 and less than 4/5 is 6/10.

Compare the two given fractions by finding a common denominator.
The lowest common denominator for 1/2 and 4/5 is 10.
Convert both fractions to equivalent fractions with a denominator of 10.
1/2 = 5/10
4/5 = 8/10
Identify a fraction between 5/10 and 8/10.
One possible fraction between these two is 6/10.
A fraction that is greater than 1/2 and less than 4/5 is 6/10.

Read more about fractions.

https://brainly.com/question/31211004

#SPJ11

Other Questions
Explain the reasons of the European Colonization of North America. the volume of air flowing into the alveoli during inspiration is increased when there is an increase in which of these? question 5 options: the pressure gradient from the atmosphere to the alveoli the volume of air in the intrapleural space airway resistance A store sells packages of comic books with a poster. A poster and 2 comics cost $7.50. A poster and 11 comics cost $16.50. Write a linear function rule that models the cost y of a package containing any number x of comic books.Suppose another store sells a similar package, modeled by a linear function rule with initial value $5.75. Use pencil and paper. Explain which store has the better deal.The linear function rule is y = a single, mature b-lymphocyte was isolated from an adult and its dna was sequenced. in the region of the genome that encodes the immunoglobulin light chain, the cell had 32 v segments and 1 j segment. how many possible variations of the light chain variable region is this cell capable of producing? read question carefully. a. 1 b. 32 c. 64 d. approximately 20,000 e. approximately 200,000 f. approximately 2,000,000 A 12 V battery is connected across two parallel metal plates separated by 0.59 cm. Find the magnitude of the electric field. If your message is specific,definite and vivid,then you are applying which principle of communication If it costs 0. 15 per square inch for the wood then what is the cost for design A and Design B Find the inverse of the function please help !!! i need this urgently. i will mark you brainlyest write an essay of 350400 words on the topic : 'every word has consequences ,every silence too.' Previous Which one of the following is not a way to improve the S/Q rating of branded pairs produced at a particular production location? Copyright ow Gold Sutan, Inc. Copying, distributing, trand party website penting som presaly pronted and contrates copyright vision o Increasing expenditures for enhanced styling/features Increasing the number of models/styles produced O Increasing expenditures for best practices training for production workers Increasing expenditures for TQM/Six Sigma programs Increasing the percentage use of superior materials How do lines 15-21 act as a flashback? what clues do they give about the rest of the story? Provide specific evidence in your response. Simplify 35x raise to the power 5 y raise to the power 3 7xy raise to the power 2 Baillie Power leased high-tech electronic equipment from Courtney Leasing on January 1, 2021. Courtney purchased the equipment from Doane Machines at a cost of $253,000, its fair value. (FV of $1, PV of $1, FVA of $1, PVA of $1, FVAD of $1 and PVAD of $1) (Use appropriate factor(s) from the tables provided. ) Related Information: Lease term 2 years (8 quarterly periods) Quarterly lease payments $18,000 at Jan. 1, 2021, and at Mar. 31, June 30, Sept. 30, and Dec. 31 thereafter Economic life of asset 5 years Interest rate charged by the lessor 8% Required: Prepare a lease amortization schedule and appropriate entries for Baillie Power from the beginning of the lease through December 31, 2021. December 31 is the fiscal year end for each company. Appropriate adjusting entries are recorded at the end of each quarter Find the exact values of sin 2. cos2u. and tan 2u using the double angle formulas Cos u =- 4/5 /2 < u < sin 2u = cos 2u= tan 2u = Find the maximum volume of a box inscribed in the tetrahedron bounded by the coordinate planes and the plane x + 1/7y + 1/6z = 1. (Use symbolic notation and fractions where needed.) the maximum volume of the box: ... Answer Please With Explanation What is the speed of light in a medium having anabsolute index of refraction of 2.3? 1) what name is given to cold sore virus while it is fused with host cell chromosome DNA? (wight letter word)2)what is the name of a virus which occurs intermittently in humans, causes cold sores near the softer tissues of the mouth?(13 letter word) As part of an exercise regimen, the probability of a person running outside is 0.45, the probability of a person joining a gym is 0.40, and the probability of a person both running outside and joining a gym is 0.25. what is the probability that a person either runs or joins a gym? emma deposits $90 into a bank that pays 4% simple interest per year. calculate the value in dollars) of her deposit after 3 years? write the correct answer.10.80